21 votos

Convergencia de la serie n=2(1)nln(n)+cos(n)

n=2(1)nln(n)+cos(n)

8 votos

Esto se mencionó como ejemplo de una serie para la que se necesita saber bastante sobre aproximaciones racionales a π para demostrar la convergencia, en un comentario a la siguiente pregunta MO. mathoverflow.net/questions/54758/

0 votos

@George, gracias por tu comentario. Me encantaría conocer los detalles si alguien aquí los conoce....

0 votos

No he visto una prueba de esto, pero yo piense en el resultado se obtendrá a partir de la equidistribución, mostrando que los términos positivos y negativos de la serie se anulan en gran medida entre sí. Probablemente tendrás que utilizar el hecho de que π tiene una medida de irracionalidad finita para obtener un límite lo suficientemente bueno como para demostrar la convergencia, como con la respuesta de David Speyer a esta pregunta anterior: math.stackexchange.com/questions/2270/

19voto

JiminyCricket Puntos 143

Esto sustituye a mi respuesta original, que era defectuosa pero llevaba en la dirección correcta (creo).

Dejaré que la serie comience en algún n02 La razón de ello se verá más adelante.

Ampliar en cos(n)/ln(n) (con |cos(n)/ln(n)|<1 para n2 ):

n=n0k=0(1)nln(n)cosk(n)lnk(n)

Intercambiemos los sumandos y preocupémonos después de si estuvo bien:

k=0n=n0(1)nln(n)cosk(n)lnk(n)

Ahora la suma interna converge según Prueba de Dirichlet . De hecho podemos proceder como en la demostración de la prueba de Dirichlet para acotar su límite (suprimiendo los índices k mientras trabajamos en fijo k ):

an=lnk1(n) bn=(1)ncosk(n)=j=0kαjkcos((j+π)n),

con |αjk|1 utilizando la fórmula de reducción de potencia para el coseno y absorbiendo el factor (1)n en los cosenos sumando π a su frecuencia.

Bm=n=n0mbn

|Bm|=|n=n0mj=0kαjkcos((j+π)n)|j=0k|n=n0mcos((j+π)n)|j=0k21cos(j+π)=:Mk

|n=n0manbn|=|Bmam+1+n=n0mBn(anan+1)| ]\[Mkam+1+n=n0mMk(anan+1)=Mkam+1+Mk(an0am+1)=Mkan0

Así, podemos aplicar la prueba comparativa con las siguientes series:

k=0Mklnk1(n0)

Esto demuestra por qué necesitamos empezar en algún n0 en lugar de 2 lo que no funcionaría ya que ln2<1 .

Ahora todo depende del comportamiento de Mk . El coseno se acerca arbitrariamente a 1 ; cuando j entra en ϵ de un múltiplo impar de π entonces 1/(1cos(j+π))1/(1(1ϵ2/2))=2/ϵ2 . Heurísticamente hablando, podríamos esperar que esto ocurriera cada 1/ϵ enteros, por lo que en promedio estos picos no destruirían la decadencia a largo plazo con lnk1(n0) (que puede hacerse cuantitativamente, aunque no cualitativamente, más fuerte aumentando n0 ). Sin embargo, no sé cómo hacer que este argumento sea riguroso, o si eso es siquiera posible dada la "imprevisibilidad" de π . Es interesante que esta prueba pueda depender de los detalles de π de esta manera. [Actualización: Gracias a George Lowther por los comentarios que apuntan a esta página de MathWorld lo que da un límite superior a la medida de irracionalidad de π (nunca lo había oído antes :-). Que esto sea finito implica que Mk puede estar limitada por una potencia de k y, por tanto, la suma exterior converge (uniformemente), creo que eso completa la demostración].

Suponiendo que se pueda resolver el problema de los picos, aún tenemos que volver a la intercambiabilidad de las sumas. Este teorema muestra que podemos intercambiar las sumas si la convergencia de

k=0n=n0m(1)nln(n)cosk(n)lnk(n)

es uniforme en m -- que lo es, ya que m se eliminó en la derivación del límite Mkan0 .

En resumen, esta derivación muestra que se puede demostrar que la serie converge si los picos irregulares en 1/(1cos(j+π)) que se producen porque j se acerca arbitrariamente a impar múltiplos de π puede demostrarse que no destruye la convergencia de la suma exterior sobre k .

Gracias por esta interesante pregunta :-).

0 votos

No entiendo cómo concluyes la última desigualdad en la línea que empieza |Bm|. Introduciendo algunas cifras aleatorias en wolframalpha se obtiene n=5100cos((1+π)n)0.689574 pero 1/(1cos(1+π))0.649223, que parece contradecir lo que has escrito cuando j=1.

0 votos

@Derek Jennings: Lo miraré más tarde esta noche; puede que se me haya escapado un factor 2 en alguna parte, pero la idea general es escribir el coseno como una suma de exponenciales y usar la fórmula para la suma parcial de una serie geométrica, así que creo que la desigualdad es correcta salvo quizás por un factor erróneo. Más adelante...

0 votos

@Derek Jennings: Tienes razón, se me ha escapado un factor 2. Gracias por darte cuenta. Si reduces las dos fracciones que obtienes de la serie geométrica a un denominador común, obtienes 8 términos de valor absoluto 1 en el numerador, y 4(1cos(j+π)) en el numerador, por lo que el límite tiene que ser 2/(1cos(j+π)) .

0voto

argv0 Puntos 502

Insertar \sum_{n=2}^{infty} (-1)^n/(ln(n)+cos(n)) en el cuadro de búsqueda de Wolfram Alpha da un valor aproximado de -4,552162944044021. Por increíble que sea la suma real, puedes estar un poco seguro de su convergencia.

0 votos

Gracias por el comentario, pero esperaba una prueba.

i-Ciencias.com

I-Ciencias es una comunidad de estudiantes y amantes de la ciencia en la que puedes resolver tus problemas y dudas.
Puedes consultar las preguntas de otros usuarios, hacer tus propias preguntas o resolver las de los demás.

Powered by:

X